Skip to content

Q3

← Back

Basic Info

Probability
└── Homework
    └── W10
        └── Q3.tex

Preview

\documentclass[12pt]{article}

\usepackage{graphicx}
\usepackage{amssymb}
\usepackage{amsmath}
\usepackage[margin=1in]{geometry}
\usepackage{fancyhdr}
\usepackage{enumerate}
\usepackage[shortlabels]{enumitem}

\pagestyle{fancy}
\fancyhead[l]{Li Yifeng}
\fancyhead[c]{Homework \#10}
\fancyhead[r]{\today}
\fancyfoot[c]{\thepage}
\renewcommand{\headrulewidth}{0.2pt}
\setlength{\headheight}{15pt}

\newcommand{\bE}{\mathbb{E}}
\newcommand{\bP}{\mathbb{P}}

\begin{document}

    \section*{Question 3}

    \noindent 
    Suppose that $X$ and $Y$ are independent and uniformly distributed on $\{0,1,2\}$.  Let
    \[
    S \;=\; X + Y,\quad
    W \;=\; X\cdot Y.
    \]

    \begin{enumerate}[start=1,label={\bfseries Part \arabic*:},leftmargin=0in]
        \bigskip\item 

        \subsection*{Solution}
        First list the probability mass function of $S$:
        \[
        \begin{array}{c|ccccc}
            s & 0 & 1 & 2 & 3 & 4\\\hline
            P(S=s) & \tfrac{1}{9} & \tfrac{2}{9} & \tfrac{3}{9} & \tfrac{2}{9} & \tfrac{1}{9}
        \end{array}
        \]
        and of $W$:
        \[
        \begin{array}{c|cccc}
            w & 0 & 1 & 2 & 4\\\hline
            P(W=w) & \tfrac{5}{9} & \tfrac{1}{9} & \tfrac{2}{9} & \tfrac{1}{9}
        \end{array}
        \]
        Therefore
        \[
        H(S)
        =-\sum_{s=0}^4 P(S=s)\,\log_2P(S=s)
        \approx2.1972\text{ bits},
        \]
        \[
        H(W)
        =-\sum_{w\in\{0,1,2,4\}}P(W=w)\,\log_2P(W=w)
        \approx1.6577\text{ bits}.
        \]

        \subsection*{Answer}
        \[
        \boxed{
            H(S)\approx2.1972\text{ bits},\quad
            H(W)\approx1.6577\text{ bits}.
        }
        \]

        \bigskip\item 

        \subsection*{Solution}
        The mutual information is
        \[
        I(S;W)
        =H(S)+H(W)-H(S,W),
        \]
        where the joint entropy
        \[
        H(S,W)
        =-\sum_{s,w}P(S=s,W=w)\,\log_2P(S=s,W=w)
        \approx2.5033\text{ bits}.
        \]
        Hence
        \[
        I(S;W)
        \approx2.1972+1.6577-2.5033
        \approx1.3516\text{ bits}.
        \]

        \subsection*{Answer}
        \[
        \boxed{I(S;W)\approx1.3516\text{ bits}.}
        \]
    \end{enumerate}

\end{document}